- Sun Aug 26, 2012 9:04 pm
#4990
This is the question in which Wirth argues that an individual can not be genetically predisposed to manic depression and Chang disputes his conclusion. The question stem asks "Which one of the following most accurately expresses Chang's criticism of Wirth's argument"
I thought this question was asking which of the answer choices most accurately expresses the method of Chang's reasoning and I got this question wrong.
However, am I right in understanding that this question is asking to describe the flaw in Wirth's argument? (that he a) presupposed that only one possibility exists where more than one can?)
Shouldn't this question be labelled a flaw question and not a method of reasoning question then??
I thought this question was asking which of the answer choices most accurately expresses the method of Chang's reasoning and I got this question wrong.
However, am I right in understanding that this question is asking to describe the flaw in Wirth's argument? (that he a) presupposed that only one possibility exists where more than one can?)
Shouldn't this question be labelled a flaw question and not a method of reasoning question then??